Sie sind auf Seite 1von 3

Homework # 5

10.6. (a).
 X
2
o
1
1
P min{X, Y } > i = P {X > i}P {Y > i} =
= i
k
2
4
n

k=i+1

Thus

o
1
P min{X, Y } i = 1 i
4
n

(b).
P {X = Y } =

P {X = i, Y = i} =

i=1

P {X = i}P {Y = i} =

i=1

X
1
1
1
1
=
=
i
1
4
414
3
i=1

(c). Notice that


P {X > Y } + P {Y > X} + P {X = Y } = 1 andP {X > Y } = P {Y > X}
Thus
P {Y > X} =

o 1
1
1 1
1 P {X = Y } =
1
=
2
2
3
3

(d).

n
[
 X
X
1 1
P X devides Y } =
P
{X = i, Y = ij} =
2i 2ij
i=1
j=1
i=1 j=1

X
i=1

1
2i(1+i)

X 1
1
1
=
2
1 2i
2i 2i 1
i=1

(e)
P {X kY } =

X
i=1

2
21+k

X
X
1 1
1 1
=2
P {X ki, Y = i} =
j
i
2 2
2i 2ki

1
1 2(1+k)

i=1

i=1 j=ki

2
21+k

10.12.
P {An i.o.} = P

\

n=1 k=n

Notice that
P

[

k=n

Ak = lim P
n


Ak P (An )
1

[

k=n

Ak

Taking limsup on the both side (notice the limit exists on the left hand side by monotonicity),

[

lim P
Ak lim sup P (An )
n

k=n

10.13. Assume complete convergence. By the first part of Borel-Cantelli lemma (the
part does not need independence).
[

\
o


P |Xn X| > i.o. = P
{|Xk X| > } = 0
n=1 k=n

Or, equivalently,


P |Xn X| eventually = 1

Thus,

lim sup |Xn X|

a.s.

Notice that > 0 can be arbitrarily small, letting 0+ on the right leads to
lim Xn = X

a.s.

()

On the other hand, assume (*) holds. By 0-1 law, X is equal to a constant almost
surely. Therefore, the sequence {Xn X} is independent.
For any > 0,
P

\

n=1 k=n

o
n
o


{|Xk X| > } = P |Xn X| > i.o. P lim Xn 6= X = 0
n

By the second part of Borel-Cantelli lemma (Here the independence is needed), we must
have

X
P {|Xn X| > } <
n=1

10. 16* First, A1 , , Ak , are independent. By Borel-Cantelli lemma. All we need


to show is that

if p 1/2
=
X
P (Ak )

k=1
<
if p < 1/2
Fix k and write

Tk =The starting time of the first consecutive head-run during


[2k , 2k+1 1] that last at least k rounds
2

Then
P (Ak ) = P {2k Tk 2k+1 k} =

2k+1
Xk

P {Tk = j}

j=2k

For j > 2k , P {Tk = j} = (1 p)pk . For j = 2k , P {Tk = j} = pk . Thus


P (Ak ) = pk + (1 p)(2k 2)pk (2p)k

(k )

Therefore, the conclusion follows from the fact that

(2p)k <

k=1

if and only if p < 1/2.


10.18. By the relation X = a Y a.s. and by Theorem 10.1-(c), X is independent of
itself. Consequently, for any number x,
P {X x} = P {X x, X x} = P {X x}2
Therefore, P {X x} = 0 or 1. Notice that the distribution function F (x) = P {X x}
is non-decreasing. There is a C such that F (x) = 0 as x < C and F (x) = 1 as x > C.
Hence, X = C a.s. Thus, Y = a C a.s.

Das könnte Ihnen auch gefallen